X-Git-Url: http://git.tremily.us/?p=course.git;a=blobdiff_plain;f=latex%2Fproblems%2FSerway_and_Jewett_8%2Fproblem03.43.tex;h=f64dd635eb36ebc42dd25979cd3b419de78e7895;hp=c3442ce946e7ff2821ac44fa9230877f915b97b2;hb=6c9b187f3a8d46ba6f8adbc338d7ea783ecf663f;hpb=2b7e6897bb83843024c55f87cc7724d66407d2db diff --git a/latex/problems/Serway_and_Jewett_8/problem03.43.tex b/latex/problems/Serway_and_Jewett_8/problem03.43.tex index c3442ce..f64dd63 100644 --- a/latex/problems/Serway_and_Jewett_8/problem03.43.tex +++ b/latex/problems/Serway_and_Jewett_8/problem03.43.tex @@ -6,7 +6,7 @@ airplane is directly above you so that the vector leading from you to it is $\vect{P}_0=7.60\E{3}\jhat\U{m}$. At $t=30.0\U{s}$, the position vector leading from you to the airplane is $\vect{P}_{30}=(8.04\E{3}\ihat+7.60\E{3}\jhat)\U{m}$ as suggested in -Figure P3.43. Determine the magnitude and orientation of the +Figure~P3.43. Determine the magnitude and orientation of the airplane's position vector at $t=45.0\U{s}$. \begin{center} \begin{asy}